1986 AJHSME Problems/Problem 25

Revision as of 23:27, 23 January 2009 by 5849206328x (talk | contribs) (New page: ==Problem== Which of the following sets of whole numbers has the largest average? <math>\text{(A)}\ \text{multiples of 2 between 1 and 101} \qquad \text{(B)}\ \text{multiples of 3 betwee...)
(diff) ← Older revision | Latest revision (diff) | Newer revision → (diff)

Problem

Which of the following sets of whole numbers has the largest average?

$\text{(A)}\ \text{multiples of 2 between 1 and 101} \qquad \text{(B)}\ \text{multiples of 3 between 1 and 101}$

$\text{(C)}\ \text{multiples of 4 between 1 and 101} \qquad \text{(D)}\ \text{multiples of 5 between 1 and 101}$

$\text{(E)}\ \text{multiples of 6 between 1 and 101}$

Solution

This problem needs a solution. If you have a solution for it, please help us out by adding it.

See Also

1986 AJHSME Problems